Problem 1298 (difficulty: 5/10)

\(\displaystyle f:\R^p\to \R^q\), \(\displaystyle A,B\subset \R^p\), \(\displaystyle x\in A\cap B\). Assume that \(\displaystyle f\) is continuous at \(\displaystyle x\) when restricted to either \(\displaystyle A\) or \(\displaystyle B\). Prove that \(\displaystyle f\) is continuous at \(\displaystyle x\) when restricted to \(\displaystyle A\cup B\). Does this remain true for a union of infinitely many sets?


Give me another random problem!

Subject, section:
Requested difficulty:
Request for a concrete problem:I want problem no.

Supported by the Higher Education Restructuring Fund allocated to ELTE by the Hungarian Government